1answer.
Ask question
Login Signup
Ask question
All categories
  • English
  • Mathematics
  • Social Studies
  • Business
  • History
  • Health
  • Geography
  • Biology
  • Physics
  • Chemistry
  • Computers and Technology
  • Arts
  • World Languages
  • Spanish
  • French
  • German
  • Advanced Placement (AP)
  • SAT
  • Medicine
  • Law
  • Engineering
posledela
3 years ago
12

Theo drinks 5/8 quart of milk with lunch and 1/4 quart of milk with dinner.

Mathematics
2 answers:
prisoha [69]3 years ago
7 0

Answer:

1/2 quart

Step-by-step explanation:

Molodets [167]3 years ago
5 0

Answer:

is 1 quart

Step-by-step explanation:

You might be interested in
Which number is greater?<br> 2.2 or 15/4 (it is a fraction)
BabaBlast [244]

Answer:

15/4

Step-by-step explanation:

15 divided by 4 = 3.75

3.75 is greater than 2.2

8 0
2 years ago
Read 2 more answers
20. Carmen can buy bottles of paint for $2.00 each and boxes of colored pencils for $3.50 each. She can spend no more than
EleoNora [17]

Answer:

(a) The equality that express many bottles of paint, x, and boxes of colored pencils, y, Carmen can buy is  2 x + 3.5 y = 42

b)Three different solutions:

x  = 14, y = 4 is First Solution.

x  = 7, y = 8 is Second Solution.

x  = 21, y = 0 is Third Solution.

Step-by-step explanation:

Here, the cost of 1 bottle of paint = $2.00 each

The cost of 1 box of colored pencils  = $3.5 each

Let us assume the number of bottle of paints purchased = x

So, the cost of x bottle of paints   = x ( Cost of 1 bottle of paint)

= x ($2.00)  = 2 x

Also, assume the number of box of colored pencils purchased = y

So, the cost of y box of colored pencils = y ( Cost of 1 box)

= y ($3.50)  = 3.5 y

Also, the total amount to be spent on art supplies  = $42

So, the total amount spent on x paint bottles + y box of colored pencil

= $42

or,  2 x + 3.5 y = 42

a ) So, the equality that express many bottles of paint, x, and boxes of colored pencils, y, Carmen can buy is  2 x + 3.5 y = 42

b)Three different solutions:

When y = 4 ,  equation is:   2 x + 3.5(4)  = 42

or,  2 x = 42 - 14  = 28, or x = 28/2  =  14

So, x  = 14, y = 4 is First Solution.

When y = 8 ,  equation is:   2 x + 3.5(8)  = 42

or,  2 x = 42 - 28  = 14, or x = 14/2  =  7

So, x  = 7, y = 8 is Second Solution.

When y = 0 ,  equation is:   2 x + 3.5(0)  = 42

or,  2 x = 42 , or x = 42/2  = 21

So, x  = 21, y = 0 is Third Solution.

7 0
3 years ago
Practice Question 2 - Find the Volume ​
Brrunno [24]

Answer:

112.896

Step-by-step explanation:

To calculate the volume of a triangular prism, measure the width and height of a triangular base, then multiply the base by the height by 1/2 to determine the triangle's area. Next, measure the height of the triangular prism and multiply this by the triangle's area to get the volume.

4 0
3 years ago
1.5 more than the quation of a and 4 is b
Kruka [31]
I think it is (a+4=b)+1.5
3 0
3 years ago
Read 2 more answers
Please help asap !!!
denpristay [2]

Answer:

B. x=3/7   y=22/7

C. because it is not precise

D. you need to replace these values in the above equation

<u>Check Understanding</u>

to find the intersection point:

y=y

-3x-2=x-10

solve, x=2

B. by graphing the 2 lines (equations)

Step-by-step explanation:

3 0
2 years ago
Other questions:
  • A city lot has the shape of a right triangle whose hypotenuse is 2 ft longer than one of the other sides. the perimeter of the l
    8·1 answer
  • Find the value of y for a given value of x, if y varies directly with x
    14·1 answer
  • The vertex of the parabola is at (4,-3) which of the following could be its equation?
    9·1 answer
  • Using the distributive property of multiplication for 12:(8+7) we get
    6·1 answer
  • I hope the picture is clear ^^<br>can anyone please help me with this
    9·1 answer
  • W is directly proportional to u if w=5 when u=3,find u when w is =2/7​
    7·1 answer
  • Solve the system of equations. y=-3/2+9 <br> 2x+2y=16
    9·1 answer
  • Simplify all of these
    14·2 answers
  • Pete will buy a computer priced at $564.20. He will use a coupon 15% off. What will be the sale price of the computer before tax
    5·2 answers
  • Pls help me with math asap pls no links thx
    5·2 answers
Add answer
Login
Not registered? Fast signup
Signup
Login Signup
Ask question!